Jump to content

NTA1907's Content

There have been 1000 items by NTA1907 (Search limited from 09-06-2020)



Sort by                Order  

#607136 $1. \begin{cases}x^{3}+2xy^{2}=y^...

Posted by NTA1907 on 04-01-2016 - 12:31 in Phương trình - hệ phương trình - bất phương trình

$3. \begin{cases}x+\frac{2xy}{\sqrt[3]{x^{2}-2x+9}}=x^{2}+y  \\  y+\frac{2xy}{\sqrt[3]{y^{2}-2y+9}}=y^{2}+x\end{cases}$

(1)+(2)$\Rightarrow x^{2}+y^{2}=2xy(\frac{1}{\sqrt[3]{(x-1)^{2}+8}}+\frac{1}{\sqrt[3]{(y-1)^{2}+8}})$
Ta có:
$\frac{1}{\sqrt[3]{(x-1)^{2}+8}}+\frac{1}{\sqrt[3]{(y-1)^{2}}+8}\leq \frac{1}{2}+\frac{1}{2}=1$
$\Rightarrow x^{2}+y^{2}\leq 2xy\Leftrightarrow x=y$
Thay vào ta được $x=y=1$ hoặc $x=y=0$



#607046 $2x^{2}-11x+21-3\sqrt[3]{4(x-1)}=0$

Posted by NTA1907 on 03-01-2016 - 21:05 in Phương trình - Hệ phương trình - Bất phương trình

Giải phương trình

$2x^{2}-11x+21-3\sqrt[3]{4(x-1)}=0$

$2x^{2}-11x+21=3\sqrt[3]{4x-4}> 0\Rightarrow x> 1$
Pt$\Leftrightarrow 2x^{2}-12x+18=3\sqrt[3]{4x-4}-(x+3)$
$\Leftrightarrow 2(x^{2}-6x+9)=\frac{-x^{3}-9x^{2}+81x-135}{9\sqrt[3]{(4x-4)^{2}}+3(x+3)\sqrt[3]{4x-4}+(x+3)^{2}}$
$\Leftrightarrow 2(x-3)^{2}+\frac{(x-3)^{2}(x+15)}{9\sqrt[3]{(4x-4)^{2}}+3(x+3)\sqrt[3]{4x-4}+(x+3)^{2}}=0$
$\Leftrightarrow (x-3)^{2}(...)=0$
$\Rightarrow x=3$ vì pt trong ngoặc luôn dương với $x> 1$



#607038 $3^{a}+2^{b}+2015=3(c!)$

Posted by NTA1907 on 03-01-2016 - 20:47 in Số học

Tìm tất cả các số nguyên a, b, c thoả mãn:

$3^{a}+2^{b}+2015=3(c!)$




#607027 Tìm giá trị nhỏ nhất của: A=$\frac{xy}{z}$...

Posted by NTA1907 on 03-01-2016 - 20:21 in Bất đẳng thức và cực trị

Cho x,y,z là các số dương;x^2+y^2+z^2=1

Tìm giá trị nhỏ nhất của:

A=$\frac{xy}{z}$+$\frac{xz}{y}$+$\frac{yz}{x}$

$A^{2}=\sum \frac{x^{2}y^{2}}{z^{2}}+2(\sum x^{2})$
Ta có:
$\frac{x^{2}y^{2}}{z^{2}}+\frac{y^{2}z^{2}}{x^{2}}\geq 2\sqrt{y^{4}}=2y^{2}$
Tương tự cộng lại ta có:
$\sum \frac{x^{2}y^{2}}{z^{2}}\geq \sum x^{2}$
$\Rightarrow A^{2}\geq 3(\sum x^{2})=3\Rightarrow A\geq \sqrt{3}$
Dấu = xảy ra$\Leftrightarrow a=b=c=\frac{1}{\sqrt{3}}$



#606983 [TOPIC] Toán nâng cao Đại số lớp 8

Posted by NTA1907 on 03-01-2016 - 16:51 in Đại số

Cho a,b,c là các số dương. Tìm GTNN của:

P=(a+b+c)($\frac{1}{a}$ + $\frac{1}{b}$ + $\frac{1}{c}$)

Vì $a, b, c> 0$ nên ta có:
$a+b+c\geq 3\sqrt[3]{abc}$
$\frac{1}{a}+\frac{1}{b}+\frac{1}{c}\geq \frac{3}{\sqrt[3]{abc}}$
Nhân 2 bđt trên ta được:
$P\geq 3\sqrt[3]{abc}.\frac{3}{\sqrt[3]{abc}}=9$
Dấu = xảy ra$\Leftrightarrow a=b=c=1$



#606949 TOPIC ôn thi Olimpic 30/04 và thi HSG toán 10

Posted by NTA1907 on 03-01-2016 - 13:56 in Thi HSG cấp Tỉnh, Thành phố. Olympic 30-4. Đề thi và kiểm tra đội tuyển các cấp.

 Bài toán(HONGKONG TST ROUND 2-BẤT ĐẲNG THỨC)

    Cho $a,b,c$ dương thỏa mãn $abc=1$.Tìm giá trị nhỏ nhất của:

$\frac{a^3+8}{a^3(b+c)}+\frac{b^3+8}{b^3(a+c)}+\frac{c^3+8}{c^3(b+a)}$ 

Ta có:
$\sum \frac{a^{3}+8}{a^{3}(b+c)}=\sum \frac{1}{b+c}+\sum \frac{8}{a^{3}(b+c)}\geq \frac{9}{2(a+b+c)}+\sum \frac{8(bc)^{2}}{a(b+c)}\geq \frac{9}{2(a+b+c)}+\frac{8(bc+ca+ab)^{2}}{2(ab+bc+ca)}=\frac{9}{2(a+b+c)}+(ab+bc+ca)+3(ab+bc+ca)\geq \frac{9}{2(a+b+c)}+\sqrt{3abc(a+b+c)}+3.3\sqrt[3]{(abc)^{2}}=\frac{9}{2(a+b+c)}+\frac{\sqrt{3(a+b+c)}}{2}+\frac{\sqrt{3(a+b+c)}}{2}+9\geq 3\sqrt[3]{\frac{27}{8}}+9=\frac{27}{2}$
Dấu = xảy ra$\Leftrightarrow a=b=c=1$



#606911 cho x,y,z dương và x+y+z=xyz

Posted by NTA1907 on 03-01-2016 - 11:05 in Bất đẳng thức và cực trị

Đề có phải là như thế này không bạn?

Cho $x, y, z> 0$ thoả mãn $x+y+z=xyz$. Tìm Max:

$P=\frac{2}{\sqrt{1+x^{2}}}+\frac{1}{\sqrt{1+y^{2}}}+\frac{1}{\sqrt{1+z^{2}}}$

 

P/s: Sao mãi không gõ được Latex vậy nhỉ?




#606906 Trình soạn thảo bị lỗi!

Posted by NTA1907 on 03-01-2016 - 10:43 in Hướng dẫn - Trợ giúp - Giải đáp thắc mắc khi sử dụng Diễn đàn

Mọi người cho mình hỏi tại sao lại không dùng được trình soạn thảo công thức toán vậy?

Máy mình cũng bị nek, sao không gõ được Latex vậy nhỉ?




#606838 $\begin{cases}x^{2} + xy - 3x +y=0 \\ x^{4} + 3x^{2}y -...

Posted by NTA1907 on 02-01-2016 - 22:57 in Phương trình - hệ phương trình - bất phương trình

$\left\{\begin{matrix} x^{2} + xy - 3x +y=0 & \\ x^{4} + 3x^{2}y - 5x^{2} + y^{2}=0 & \end{matrix}\right.$

Pt(1)$\Leftrightarrow x^{2}+y=3x-xy$(3) 

Pt(2)$\Leftrightarrow (x^{2}+y)^{2}+x^{2}y-5x^{2}=0$(4)

Thay (3) vào (4) ta dc:

$x^{2}(3-y)^{2}+x^{2}y-5x^{2}=0 \Leftrightarrow x^{2}(y-1)(y-4)=0$

Đến đây dễ rồi




#606812 $x^2+x+12\sqrt{x+1}=36$

Posted by NTA1907 on 02-01-2016 - 21:34 in Phương trình - hệ phương trình - bất phương trình

Giải pt
2) $x^2+x+12\sqrt{x+1}=36$

ĐK: $x\geq -1$

Pt$\Leftrightarrow 12(\sqrt{x+1}-2)+x^{2}+x-12=0$

$\Leftrightarrow 12.\frac{x-3}{\sqrt{x+1}+2}+(x-3)(x+4)=0$

$\Leftrightarrow (x-3)(\frac{12}{\sqrt{x+1}+2}+x+4)=0$

$\Leftrightarrow x=3$ hoặc $\frac{12}{\sqrt{x+1}+2}+x+4=0$(*)

Mà VT của (*) luôn dương với $x\geq -1$

$\Rightarrow x=3$ là nghiệm duy nhất của pt




#606765 CM $\frac{1}{\sqrt{x^{2}+1}...

Posted by NTA1907 on 02-01-2016 - 17:54 in Bất đẳng thức và cực trị

CM $\frac{1}{\sqrt{x^{2}+1}}+\frac{1}{\sqrt{y^{2}+1}} \leq \frac{2}{\sqrt{1+xy}}$ với x,y>=0, xy<=1

Ta có:

$\frac{1}{2}(\frac{1}{\sqrt{1+y^{2}}}+\frac{1}{\sqrt{1+x^{2}}})^{2}\leq \frac{1}{1+y^{2}}+\frac{1}{1+x^{2}}=1+\frac{1-y^{2}x^{2}}{(1+y^{2})(1+x^{2})}\leq 1+\frac{1-y^{2}x^{2}}{(1+yx)^{2}}=\frac{2}{1+yx}$

$\Rightarrow$ đpcm




#606760 $P=\sqrt{\frac{a}{b+c}}+\sq...

Posted by NTA1907 on 02-01-2016 - 17:12 in Bất đẳng thức - Cực trị

Cho $a, b, c> 0$. Tìm Min:

$P=\sqrt{\frac{a}{b+c}}+\sqrt[3]{\frac{b}{c+a}}+\sqrt[4]{\frac{c}{a+b}}$




#606559 CMR:$\frac{x}{\sqrt[3]{yz}}+...

Posted by NTA1907 on 01-01-2016 - 19:33 in Bất đẳng thức và cực trị

1.Cho các số thực dương $x,y,z$ thỏa mãn $x^2+y^2+z^2=3$.

CMR:$\frac{x}{\sqrt[3]{yz}}+\frac{y}{\sqrt[3]{xz}}+\frac{z}{\sqrt[3]{xy}}\geq xy+yz+xz$

Ta có:
$\sum \frac{x}{\sqrt[3]{yz}}=\sum \frac{x\sqrt[3]{x}}{\sqrt[3]{xyz}}$
Mà $3=x^{2}+y^{2}+z^{2}\geq 3\sqrt[3]{(xyz)^{2}}\Rightarrow \sqrt[3]{xyz}\leq 1$
$\Rightarrow \frac{x}{\sqrt[3]{yz}}\geq \sum x\sqrt[3]{x}$
Ta có:
$(\sum x\sqrt[3]{x})(\sum \sqrt[3]{x^{2}})\geq (x+y+z)^{2}\geq 3(xy+yz+zx)$
$\Rightarrow \sum x\sqrt[3]{x}\geq \frac{3(xy+yz+zx)}{\sum \sqrt[3]{x^{2}}}$
Mà $\sum \sqrt[3]{x^{2}.1.1}\leq \frac{x^{2}+y^{2}+z^{2}+6}{3}=3$
$\Rightarrow$ đpcm



#606558 Tìm Max $F = \frac{\sqrt{ab}}{a+b+2c...

Posted by NTA1907 on 01-01-2016 - 19:28 in Bất đẳng thức và cực trị

Đấy là tài liệu gì vậy ạ

không biết nữa mình lấy từ 1 bài viết bên AoPS




#606551 Tìm Max $F = \frac{\sqrt{ab}}{a+b+2c...

Posted by NTA1907 on 01-01-2016 - 19:08 in Bất đẳng thức và cực trị

Cho $a,b,c > 0$. Tìm Max

$F = \frac{\sqrt{ab}}{a+b+2c} + \frac{\sqrt{bc}}{b+c+2a} + \frac{\sqrt{ca}}{c+a+2b}$

Tương tự như bài này

918a28593aab437fc1f9a67bab5e2feaf2db6d46.jpg

d5389607c001daec91cb4436a4050bc38ebc3df9.jpg




#606515 CM các bất đẳng thức sau :

Posted by NTA1907 on 01-01-2016 - 17:35 in Bất đẳng thức và cực trị

Bạn sử dụng kiến thức trong chương trình phổ cập GD THCS giúm mình với , mình chưa học qua sigma :(

$\sum a=a+b+c$

Tương tự mấy cái còn lại




#606511 CM các bất đẳng thức sau :

Posted by NTA1907 on 01-01-2016 - 17:20 in Bất đẳng thức và cực trị

1. Cho a,b,c là các số thực dương. Thỏa mãn abc=1. CMR $\frac{a}{(ab+a+1)^{2}} + \frac{b}{(bc+b+1)^{2}} + \frac{c}{(ca+c+1)^{2}} \geq \frac{1}{a+b+c}$

 

2. Cho a,b,c > 0. CMR $\frac{a^{2}}{\sqrt{3a^{2}+8b^{2}+14ab}} + \frac{b^{2}}{\sqrt{3b^{2}+8c^{2}+14bc}} + \frac{c^{2}}{\sqrt{3c^{2}+8a^{2}+14ca}} \geq \frac{a+b+c}{5}$

1.

Áp dụng bđt Bu-nhi-a-cốp-xki ta có:

$\left [ \sum \frac{a}{(ab+a+1)^{2}} \right ](\sum a)\geq (\sum \frac{a}{ab+a+1})$

Mà $abc=1$ nên $\sum \frac{a}{ab+a+1}=1$(chứng minh dễ dàng)

$\Rightarrow đpcm$

Dấu = xảy ra$\Leftrightarrow a=b=c=1$

2.

Ta có:

$\sqrt{3a^{2}+8b^{2}+14ab}=\sqrt{(3a+2b)(a+4b)}\leq \frac{3a+2b+a+4b}{2}=2a+3b$

$\Rightarrow \frac{a^{2}}{\sqrt{3a^{2}+8b^{2}+14ab}}\geq \frac{a^{2}}{2a+3b}$

$\Rightarrow VT\geq \sum \frac{a^{2}}{2a+3b}\geq \frac{(a+b+c)^{2}}{5(a+b+c)}=\frac{a+b+c}{5}$

Dấu = xảy ra$\Leftrightarrow a=b=c$




#606503 $P=\frac{a-b}{c}+\frac{b-c}...

Posted by NTA1907 on 01-01-2016 - 17:04 in Bất đẳng thức và cực trị

Cho a,b,c là các số thực dương thỏa mãn: $min\left \{ a,b,c \right \}\geq \frac{1}{4}max\left \{ a,b,c \right \}$.

Tìm giá trị nhỏ nhất và giá trị lớn nhất của biếu thức: $P=\frac{a-b}{c}+\frac{b-c}{a}+\frac{c-a}{b}$

P/s: Mọi người làm nhanh kẻo trôi ạ  :D  :D  

hhhhhh.JPG

P/s: ai dịch được không?




#606298 $\boxed{\text{Chuyên Đề}}$ Phương trình vô tỉ - Hệ phương...

Posted by NTA1907 on 31-12-2015 - 14:26 in Phương trình, hệ phương trình và bất phương trình

8/     $\sqrt{3x^{2}+6x+7}+\sqrt{5x^{2}+10x+14}=4-2x-x^{2}$

$VT=\sqrt{3(x+1)^{2}+4}+\sqrt{5(x+1)^{2}+9}\geq \sqrt{4}+\sqrt{9}=5$

$VP=5-(x+1)^{2}\leq 5$

$VT=VP\Leftrightarrow x=-1$




#606221 \left\{\begin{matrix} x^3+3xy^2=-49 &...

Posted by NTA1907 on 30-12-2015 - 21:56 in Phương trình - hệ phương trình - bất phương trình

$\left\{\begin{matrix} x^3+3xy^2=-49 & \\x^2-8xy+y^2=8y-17x^2 & \end{matrix}\right.$

Lấy pt(1)+3.pt(2) ta được:

$x^{3}+3xy^{2}+49+3(x^{2}-8xy+y^{2}-8y+17x)=0$

$\Leftrightarrow (x^{3}+3x^{2}+3x+1)+3(x+1)(y^{2}-8y+16)=0$

$\Leftrightarrow (x+1)^{3}+3(x+1)(y-4)^{2}=0$

$\Leftrightarrow (x+1)\left [ (x+1)^{2}+3(y-4)^{2} \right ]=0$

Đến đây dễ rồi




#606207 $x^{3}+3x^{2}+4x=\sqrt{2x-1}-1$

Posted by NTA1907 on 30-12-2015 - 21:38 in Phương trình, hệ phương trình và bất phương trình

Giải phương trình

$x^{3}+3x^{2}+4x=\sqrt{2x-1}-1$

ĐK: $x\geq \frac{1}{2}$

Pt$\Leftrightarrow (x+1)^{3}+x-\sqrt{2x-1}=0$

$\Leftrightarrow (x+1)^{3}+\frac{(x-1)^{2}}{x+\sqrt{2x-1}}=0$(1)

Vì $x\geq \frac{1}{2}$ nên VT của (1) luôn $> 0\Rightarrow Vô nghiệm$




#606116 $ab-bc+ca\leq 2$

Posted by NTA1907 on 30-12-2015 - 13:46 in Bất đẳng thức và cực trị

Cho $a, b, c\geq 0$ thoả mãn $a+2b+3c=4$. CMR: $ab-bc+ca\leq 2$




#606114 $ 16x^2+10x+1=\sqrt{2x+3} $

Posted by NTA1907 on 30-12-2015 - 13:20 in Phương trình, hệ phương trình và bất phương trình

Giải phương trình: 

$ 16x^2+10x+1=\sqrt{2x+3} $

theo mình thì đề phải là "-" mới đúng 




#606044 $\left\{\begin{matrix} &\overline...

Posted by NTA1907 on 29-12-2015 - 22:24 in Số học

Tìm tất cả các số tự nhiên có ba chữ số $\overline{abc}$ sao cho: $\left\{\begin{matrix} &\overline{abc}=n^{2}-1 & \\ &\overline{cba}=(n-2)^{2} & \end{matrix}\right.$ với n là số nguyên lớn hơn 2

$\left\{\begin{matrix} &100a+10b+c=n^{2}-1 \\ &100c+10b+a=n^{2}-4n+4 \end{matrix}\right.$ 

$\Rightarrow 99(a-c)=4n-5\vdots 99$ 

$\Rightarrow 4n-5\geq 99\Leftrightarrow n\geq 26$ 

Từ pt(1)$\Rightarrow n\leq 31$ 

Từ pt(2)$\Rightarrow n\leq 33$ 

$\Rightarrow 26\leq n\leq 31$ Đến đây thay vào là được




#606032 $\frac{\sqrt{(1+b^{2}c^{2})(1+a^...

Posted by NTA1907 on 29-12-2015 - 22:05 in Đại số

Cho 3 số dương $a$,$b$,$c$ thỏa mãn điều kiện: $a+b+c=\frac{1}{abc}$

Chứng minh rằng: $\frac{\sqrt{(1+b^{2}c^{2})(1+a^{2}c^{2})}}{\sqrt{c^{2}+a^{2}b^{2}c^{2}}}=a+b$

$\frac{\sqrt{(1+b^{2}c^{2})(1+a^{2}c^{2})}}{\sqrt{c^{2}(1+a^{2}b^{2})}}$

Thay $1=abc(a+b+c)$ vào rồi phân tích thành nhân tử